Prove Inequality n! > n^3 for n > 5 w/ M.I.

And no I don't believe it, but it doesn't matter, I'm just proving that through the transitive property of inequalities that k^4 > 3k^2 + 3k + 1 > k^3(k+1) > (k+1)^3. I'm just trying to prove that k^4 > (k+1)^3, but it seems impossible without a proof by induction.
  • #1
Panphobia
435
13

Homework Statement


Use M.I. to prove that n! > n^3 for n > 5

The Attempt at a Solution



I already proved n! > n^2 for n>4, but this is nothing like that.
This is my inductive step so far.
n=k+1
(k+1)! > (k+1)^3

(k+1)! - (k+1)^3 > 0[/B]

(k+1)! - (k+1)^3 = (k+1)[k! - (k+1)^2]
> (k+1)[k^3 - (k+1)^2]

This is how far I can get, any hints? Am I going in the right track or am I going in the wrong direction completely?
 
Physics news on Phys.org
  • #2
Panphobia said:

Homework Statement


Use M.I. to prove that n! > n^3 for n > 5

The Attempt at a Solution



I already proved n! > n^2 for n>4, but this is nothing like that.
This is my inductive step so far.
n=k+1
(k+1)! > (k+1)^3[/B]
Don't start off with this - it's what you want to show. Use your assumption (i.e., that k! > k^3) and write (k + 1)! as (k + 1)k!
Panphobia said:

(k+1)! - (k+1)^3 > 0


(k+1)! - (k+1)^3 = (k+1)[k! - (k+1)^2]
> (k+1)[k^3 - (k+1)^2]

This is how far I can get, any hints? Am I going in the right track or am I going in the wrong direction completely?
 
  • #3
Mark44 said:
Don't start off with this - it's what you want to show. Use your assumption (i.e., that k! > k^3) and write (k + 1)! as (k + 1)k!
I only started it that way because I proved n! > n^2 the same exact way. Hmmm I am thinking about this at the moment.
 
  • #4
So then if k! > k^3, then (k+1)! > k^3(k+1), is this going on the right track, if it is I don't know what else to do. Wait, then do I have to prove that k^3(k+1) >= (k+1)^3 where k>5?
 
  • #5
Panphobia said:
So then if k! > k^3, then (k+1)! > k^3(k+1), is this going on the right track,
No. You assume that k! > k^3, and then you use that assumption to show (prove) that (k + 1)! > (k + 1)^3.
Panphobia said:
if it is I don't know what else to do. Wait, then do I have to prove that k^3(k+1) >= (k+1)^3 where k>5?

Your work for that step should start off like this:
(k + 1)! = (k + 1) k! > (k + 1) k^3

Work with that last expression to show that it is larger than (k + 1)^3, at least when k > 5.
 
  • #6
I give up haha, I can't figure it out on my own, I know intuitively that (k+1)*k^3 > (k+1)^3, but don't know any way to prove it. I tried dividing out the (k+1) then leaving it as k^3 > (k+1)^2 then expanding (k+1)^2, but that was a dead end.
 
  • #7
Panphobia said:
I give up haha, I can't figure it out on my own, I know intuitively that (k+1)*k^3 > (k+1)^3, but don't know any way to prove it. I tried dividing out the (k+1) then leaving it as k^3 > (k+1)^2 then expanding (k+1)^2, but that was a dead end.
You can't "divide out" anything, since you're not working with an equation or inequality. The last expression I showed in my previous post was (k + 1)k^3, which is the same as (equal to) k^4 + k^3. Intuitively, that should be larger than (k + 1)^3, being that the first is a 4th degree polynomial and the second is only a cubic.
 
  • #8
Yea, k^4 + k^3 > k^3 + 3k^2 + 3k + 1
k^4 > 3k^2 + 3k + 1

3k^2 + 3k + 1 < 3k^2 + 3k^2 + k^2 < 7k^2 < k^4

So k^3(k+1) > (k+1)^3
 
  • #9
Panphobia said:
Yea, k^4 + k^3 > k^3 + 3k^2 + 3k + 1
Yeah, but you need to show why this is true, not just wave your arms. Possibly it might require a separate proof by induction.
Panphobia said:
k^4 > 3k^2 + 3k + 1

3k^2 + 3k + 1 < 3k^2 + 3k^2 + k^2 < 7k^2 < k^4

So k^3(k+1) > (k+1)^3
 
  • #10
Yea that's what I thought, is there no other simpler way to prove this?
 
  • #11
k^4 > 3k^2 + 3k + 1 is equivalent to k^4 - 3k^2 - 3k - 1 > 0. It might be acceptable to show that the graph of y = x^4 - 3x^2 - 3x - 1 lies above the x-axis for all x larger than a specific x-value.
 
  • #12
Panphobia said:
Yea, k^4 + k^3 > k^3 + 3k^2 + 3k + 1
k^4 > 3k^2 + 3k + 1

3k^2 + 3k + 1 < 3k^2 + 3k^2 + k^2 < 7k^2 < k^4

So k^3(k+1) > (k+1)^3
Do you really believe that 3k^2 + 3k^2 + k^2 < 7k^2? Saying things like that is a sure way to lose marks on an exam.
 
  • #13
Ray Vickson said:
Do you really believe that 3k^2 + 3k^2 + k^2 < 7k^2? Saying things like that is a sure way to lose marks on an exam.
On my sheet of paper its an equals sign, typo.
 

Related to Prove Inequality n! > n^3 for n > 5 w/ M.I.

1. What is the purpose of using mathematical induction to prove this inequality?

The purpose of using mathematical induction is to prove that the given statement holds true for all values of n greater than 5. It is a powerful method for proving mathematical statements and involves a step-by-step process of proving the statement for a base case and then showing that if the statement holds for a certain value of n, it also holds for the next value of n.

2. What is the base case for this mathematical induction?

The base case for this mathematical induction is n = 6. This means that we will first prove that the statement n! > n^3 is true for n = 6. Once we have proven this, we will then move on to proving that if the statement holds for n = 6, it also holds for n = 7 and so on.

3. How does the inductive step work in this proof?

The inductive step involves assuming that the statement is true for a certain value of n (in this case, n = k) and then using this assumption to prove that the statement also holds for the next value of n (in this case, n = k+1). This is done by substituting n = k+1 into the statement and manipulating the inequality until it is equivalent to the statement for n = k. If this can be done, then the statement holds true for all values of n greater than k.

4. What is the key factor in proving this inequality?

The key factor in proving this inequality is understanding the properties of factorial and exponents. It is important to note that n! is equal to the product of all positive integers from 1 to n, and n^3 is equal to n multiplied by itself three times. By using these properties, we can manipulate the inequality to prove that n! is always greater than n^3 for n greater than 5.

5. Are there any exceptions to this inequality?

No, there are no exceptions to this inequality. This statement holds true for all values of n greater than 5. This can be proven using mathematical induction, which shows that if the statement is true for a certain value of n, it will also be true for the next value of n. Therefore, n! will always be greater than n^3 for values of n greater than 5.

Similar threads

  • Precalculus Mathematics Homework Help
Replies
6
Views
1K
  • Precalculus Mathematics Homework Help
Replies
1
Views
1K
  • Precalculus Mathematics Homework Help
Replies
4
Views
808
  • Precalculus Mathematics Homework Help
Replies
14
Views
1K
  • Precalculus Mathematics Homework Help
Replies
13
Views
1K
  • Precalculus Mathematics Homework Help
Replies
6
Views
1K
  • Precalculus Mathematics Homework Help
Replies
6
Views
1K
  • Precalculus Mathematics Homework Help
Replies
8
Views
1K
  • Precalculus Mathematics Homework Help
Replies
2
Views
943
  • Precalculus Mathematics Homework Help
Replies
6
Views
853
Back
Top